Double or Take gameRook game on chessboardGuess my number! Pay for the answer!The subtraction gameKnight on a 5 by 5 boardDim - A Game with PebblesFace Up Poker with Alice and BobAlternate Face Up Poker with Alice and Bob (on the floor)Let's revisit this later (a two player knight's game)Rook Game on a Chessboard - Take 2Paper, pencil and a bunch of bars

How does Howard Stark know this?

Ex-manager wants to stay in touch, I don't want to

Why does a C.D.F need to be right-continuous?

How can a Lich look like a human without magic?

Renting a house to a graduate student in my department

Why is it so slow when assigning a concatenated string to a variable in python?

How to make the table in the figure in LaTeX?

Is Simic Ascendancy triggered by Awakening of Vitu-Ghazi?

Should these notes be played as a chord or one after another?

Why was this sacrifice sufficient?

Best species to breed to intelligence

How old is Captain America at the end of "Avengers: Endgame"?

Would an 8% reduction in drag outweigh the weight addition from this custom CFD-tested winglet?

Why do Thanos's punches not kill Captain America or at least cause some mortal injuries?

How to make a language evolve quickly?

Why use steam instead of just hot air?

On what legal basis did the UK remove the 'European Union' from its passport?

Why in a Ethernet LAN, a packet sniffer can obtain all packets sent over the LAN?

How can this pool heater gas line be disconnected?

Remove everything except csv file Bash Script

International Code of Ethics for order of co-authors in research papers

How do I get past a 3-year ban from overstay with VWP?

Set a camera to free fall like a Rigid Body?

What does it mean with the ask price is below the last price?



Double or Take game


Rook game on chessboardGuess my number! Pay for the answer!The subtraction gameKnight on a 5 by 5 boardDim - A Game with PebblesFace Up Poker with Alice and BobAlternate Face Up Poker with Alice and Bob (on the floor)Let's revisit this later (a two player knight's game)Rook Game on a Chessboard - Take 2Paper, pencil and a bunch of bars













8












$begingroup$


Double or Take is a two-player number game. Alice starts by selecting any positive integer. Bob's options are to:



  • subtract a positive perfect square

  • subtract a positive perfect cube, or

  • double the number.

Alice then takes the result, and has the same options. Play alternates, with the winner being the first player to reach exactly zero.



What are the possible starting numbers between $1$ and $11$ that Alice could choose to be sure that she can beat Bob?



And as a bonus (it's not essential to answer this, and may be tedious with pencil and paper): can Alice be sure of beating Bob if she chooses $12$ as the starting number?










share|improve this question











$endgroup$











  • $begingroup$
    Source for the game: Martin Gardner in a 1970's Sci.Am. column, I think, though I'm now unable to locate it. If anyone can find the exact column I'd be very grateful.
    $endgroup$
    – MichaelMaggs
    May 1 at 15:16










  • $begingroup$
    "reaching zero" implies exactly zero? As in not going beyond zero to a negative integer?
    $endgroup$
    – BDrought
    May 1 at 15:28










  • $begingroup$
    That's right. Players are not allowed to end up with a negative number.
    $endgroup$
    – MichaelMaggs
    May 1 at 15:30










  • $begingroup$
    I do have the answer for 12, but as there are a lot of possible plays it would be lengthy to set them all out in full. And maybe not that interesting.
    $endgroup$
    – MichaelMaggs
    May 1 at 15:47











  • $begingroup$
    I'm still looking at the situation starting at 12, and you're not wrong about it being tedious with pencil and paper. How do you feel about computer-assisted solutions to this one?
    $endgroup$
    – Gareth McCaughan
    May 1 at 16:21















8












$begingroup$


Double or Take is a two-player number game. Alice starts by selecting any positive integer. Bob's options are to:



  • subtract a positive perfect square

  • subtract a positive perfect cube, or

  • double the number.

Alice then takes the result, and has the same options. Play alternates, with the winner being the first player to reach exactly zero.



What are the possible starting numbers between $1$ and $11$ that Alice could choose to be sure that she can beat Bob?



And as a bonus (it's not essential to answer this, and may be tedious with pencil and paper): can Alice be sure of beating Bob if she chooses $12$ as the starting number?










share|improve this question











$endgroup$











  • $begingroup$
    Source for the game: Martin Gardner in a 1970's Sci.Am. column, I think, though I'm now unable to locate it. If anyone can find the exact column I'd be very grateful.
    $endgroup$
    – MichaelMaggs
    May 1 at 15:16










  • $begingroup$
    "reaching zero" implies exactly zero? As in not going beyond zero to a negative integer?
    $endgroup$
    – BDrought
    May 1 at 15:28










  • $begingroup$
    That's right. Players are not allowed to end up with a negative number.
    $endgroup$
    – MichaelMaggs
    May 1 at 15:30










  • $begingroup$
    I do have the answer for 12, but as there are a lot of possible plays it would be lengthy to set them all out in full. And maybe not that interesting.
    $endgroup$
    – MichaelMaggs
    May 1 at 15:47











  • $begingroup$
    I'm still looking at the situation starting at 12, and you're not wrong about it being tedious with pencil and paper. How do you feel about computer-assisted solutions to this one?
    $endgroup$
    – Gareth McCaughan
    May 1 at 16:21













8












8








8





$begingroup$


Double or Take is a two-player number game. Alice starts by selecting any positive integer. Bob's options are to:



  • subtract a positive perfect square

  • subtract a positive perfect cube, or

  • double the number.

Alice then takes the result, and has the same options. Play alternates, with the winner being the first player to reach exactly zero.



What are the possible starting numbers between $1$ and $11$ that Alice could choose to be sure that she can beat Bob?



And as a bonus (it's not essential to answer this, and may be tedious with pencil and paper): can Alice be sure of beating Bob if she chooses $12$ as the starting number?










share|improve this question











$endgroup$




Double or Take is a two-player number game. Alice starts by selecting any positive integer. Bob's options are to:



  • subtract a positive perfect square

  • subtract a positive perfect cube, or

  • double the number.

Alice then takes the result, and has the same options. Play alternates, with the winner being the first player to reach exactly zero.



What are the possible starting numbers between $1$ and $11$ that Alice could choose to be sure that she can beat Bob?



And as a bonus (it's not essential to answer this, and may be tedious with pencil and paper): can Alice be sure of beating Bob if she chooses $12$ as the starting number?







game number-theory nim






share|improve this question















share|improve this question













share|improve this question




share|improve this question








edited May 1 at 21:11









Hugh

2,34311127




2,34311127










asked May 1 at 15:16









MichaelMaggsMichaelMaggs

47211




47211











  • $begingroup$
    Source for the game: Martin Gardner in a 1970's Sci.Am. column, I think, though I'm now unable to locate it. If anyone can find the exact column I'd be very grateful.
    $endgroup$
    – MichaelMaggs
    May 1 at 15:16










  • $begingroup$
    "reaching zero" implies exactly zero? As in not going beyond zero to a negative integer?
    $endgroup$
    – BDrought
    May 1 at 15:28










  • $begingroup$
    That's right. Players are not allowed to end up with a negative number.
    $endgroup$
    – MichaelMaggs
    May 1 at 15:30










  • $begingroup$
    I do have the answer for 12, but as there are a lot of possible plays it would be lengthy to set them all out in full. And maybe not that interesting.
    $endgroup$
    – MichaelMaggs
    May 1 at 15:47











  • $begingroup$
    I'm still looking at the situation starting at 12, and you're not wrong about it being tedious with pencil and paper. How do you feel about computer-assisted solutions to this one?
    $endgroup$
    – Gareth McCaughan
    May 1 at 16:21
















  • $begingroup$
    Source for the game: Martin Gardner in a 1970's Sci.Am. column, I think, though I'm now unable to locate it. If anyone can find the exact column I'd be very grateful.
    $endgroup$
    – MichaelMaggs
    May 1 at 15:16










  • $begingroup$
    "reaching zero" implies exactly zero? As in not going beyond zero to a negative integer?
    $endgroup$
    – BDrought
    May 1 at 15:28










  • $begingroup$
    That's right. Players are not allowed to end up with a negative number.
    $endgroup$
    – MichaelMaggs
    May 1 at 15:30










  • $begingroup$
    I do have the answer for 12, but as there are a lot of possible plays it would be lengthy to set them all out in full. And maybe not that interesting.
    $endgroup$
    – MichaelMaggs
    May 1 at 15:47











  • $begingroup$
    I'm still looking at the situation starting at 12, and you're not wrong about it being tedious with pencil and paper. How do you feel about computer-assisted solutions to this one?
    $endgroup$
    – Gareth McCaughan
    May 1 at 16:21















$begingroup$
Source for the game: Martin Gardner in a 1970's Sci.Am. column, I think, though I'm now unable to locate it. If anyone can find the exact column I'd be very grateful.
$endgroup$
– MichaelMaggs
May 1 at 15:16




$begingroup$
Source for the game: Martin Gardner in a 1970's Sci.Am. column, I think, though I'm now unable to locate it. If anyone can find the exact column I'd be very grateful.
$endgroup$
– MichaelMaggs
May 1 at 15:16












$begingroup$
"reaching zero" implies exactly zero? As in not going beyond zero to a negative integer?
$endgroup$
– BDrought
May 1 at 15:28




$begingroup$
"reaching zero" implies exactly zero? As in not going beyond zero to a negative integer?
$endgroup$
– BDrought
May 1 at 15:28












$begingroup$
That's right. Players are not allowed to end up with a negative number.
$endgroup$
– MichaelMaggs
May 1 at 15:30




$begingroup$
That's right. Players are not allowed to end up with a negative number.
$endgroup$
– MichaelMaggs
May 1 at 15:30












$begingroup$
I do have the answer for 12, but as there are a lot of possible plays it would be lengthy to set them all out in full. And maybe not that interesting.
$endgroup$
– MichaelMaggs
May 1 at 15:47





$begingroup$
I do have the answer for 12, but as there are a lot of possible plays it would be lengthy to set them all out in full. And maybe not that interesting.
$endgroup$
– MichaelMaggs
May 1 at 15:47













$begingroup$
I'm still looking at the situation starting at 12, and you're not wrong about it being tedious with pencil and paper. How do you feel about computer-assisted solutions to this one?
$endgroup$
– Gareth McCaughan
May 1 at 16:21




$begingroup$
I'm still looking at the situation starting at 12, and you're not wrong about it being tedious with pencil and paper. How do you feel about computer-assisted solutions to this one?
$endgroup$
– Gareth McCaughan
May 1 at 16:21










3 Answers
3






active

oldest

votes


















8












$begingroup$

Answer to the non-bonus part:




Next-player winning positions -- i.e., ones where Bob wins if he starts -- from 1 to 11 inclusive are 1,3,4,6,8,9,10,11. Next-player losing positions are 2,5,7. So Alice (who plays second) wins if she chooses 2, 5, or 7, but not if she chooses any other number from 1 to 11.




Proof:




Call a position "winning" or "losing" according to what happens with best play to the player whose move it is. (Or "drawing" if neither player can force a win and at least one can force the game to continue for ever.) Then we find, successively:

0 is losing (whoever just moved there already won the game).

1,4,8,9 are winning (you can move to 0 from any of them).

2 is losing (you can move from there only to 1,4, both of which are winning).

3,6,10 are winning (you can move to 2 from any of them).

5 is losing (you can move from there only to 1,4,10, all of which are winning).

14 is winning (you can move to 5 from it).

7 is losing (you can move from there only to 3,6,14, all of which are winning).

11 is winning (you can move to 7 from it).




Bonus part: From 12




it turns out -- I calculated this by computer -- that the sort of reasoning above is enough to determine that all positions you can get to from 12 other than 24 are next-player wins, and all positions you can get to from 24 other than 48 are next-player wins. Now, we have $12rightarrow24rightarrow48rightarrow12$. So, if Bob loses starting from 12 in particular he must lose when he moves to 24; if Alice wins starting at 24 then it must be by moving to 48 (since all her other moves from 24 lose); if Bob loses when starting at 48 then in particular he must lose by moving to 12, i.e., Alice must win when starting from 12. But this is a contradiction since we started out by supposing that 12 is a loss for the player moving there.




Hence




Alice cannot be sure of winning if she makes Bob move first from 12. He will move to 24, Alice definitely loses if she moves to anything other than 48, and then either Bob can win by doing something other than moving back to 12 (in which case he should) or he can't (in which case he should move back to 12, and then the best Alice can do is to go around the same loop again).







share|improve this answer











$endgroup$












  • $begingroup$
    I didn’t look at this answer when composing mine below, but I came to the same conclusion as @MichaelMaggs. Sorry, Gareth!
    $endgroup$
    – El-Guest
    May 1 at 15:40










  • $begingroup$
    @El-Guest Not sure what you're apologizing for.
    $endgroup$
    – Gareth McCaughan
    May 1 at 15:41










  • $begingroup$
    @GarethMcCaughan for (potentially) nipping your answer.
    $endgroup$
    – El-Guest
    May 1 at 15:45










  • $begingroup$
    Oh! I confess it hadn't occurred to me that that was a danger; I had an essentially correct answer some time before yours, after all. But of course OP can award the checkmark on whatever basis he pleases...
    $endgroup$
    – Gareth McCaughan
    May 1 at 15:50






  • 1




    $begingroup$
    ... I've thrown away the bits of paper on which I did some hand calculations and restarted the computer on which I did some not-hand calculations without saving the relevant stuff. I might do it again and see whether indeed the other moves from 24 can be eliminated -- I think they might be.
    $endgroup$
    – Gareth McCaughan
    May 7 at 8:33


















4












$begingroup$

Let’s see: I assume that all numbers must be greater than or equal to zero, otherwise the possibilities become too painful. If that was the intention, I’ll look harder.



1




Bob subtracts 1, so Alice loses.




2




Bob can’t double, because that leads to 4 and a Bob-loss; he can’t subtract 1 because that leads to 1 and a Bob-loss. Therefore 2 is a winning number for Alice.




3




Bob subtracts 1, forcing a loss for Alice as above.




4




Bob subtracts 4 and Alice loses.




5




Bob can’t subtract either 1 or 4, since Alice can win. He also can’t double, because that makes 10 — then Alice subtracts 8 forcing a Bob-loss. Therefore 5 is a winning number for Alice.




6




Bob subtracts 4, forcing a loss for Alice as 2 is left.




7




If Bob subtracts 1, Alice subtracts 4, forcing a loss for Bob. If Bob subtracts 4, Alice subtracts 1, forcing a loss for Bob. Bob therefore has to double to 14. But Bob doubling to 14 means Alice can subtract 9 to get to 5, which forces a Bob-loss. Thus 7 is a winning number for Alice.




8




Bob subtracts 8 ($2^3$) and Alice loses.




9




Bob subtracts 4, forcing a loss for Alice as 5 is left.




10




Bob subtracts 8, leading to an Alice loss.




11




Bob subtracts 4, leading to an Alice loss.




Therefore the only starting numbers that win for Alice are




2, 5, and 7.




Bonus Part: 12




Bob has to double to 24 because he can’t get to 2, 5, or 7. Then Alice can’t subtract 1, because it’ll leave Bob with 23; he can subtract 16 and win. Alice can’t subtract 9, because it’ll leave Bob with 15; he can subtract 8 and win. Alice can’t subtract 8, because it’ll leave Bob with 16; he can subtract 9 and win. Alice can’t subtract 16, because it’ll leave Bob with 8; he can subtract 1 and win. Doubling here allows Bob to subtract 36 and leave Alice with 12. It appears then that this could create a stable loop solution where neither player wins. If Alice subtracts 4, Bob is left with 20. Bob can’t subtract 4, because leaving 16 allows Alice to subtract 9 and get to 7. Bob can’t subtract 9, because leaving 11 allows Alice to subtract 4 and get to 7. Bob can’t subtract 16 because it leaves 4 for Alice. However, subtracting 8 leaves an equilibrium standoff solution. This might be optimal...?







share|improve this answer











$endgroup$












  • $begingroup$
    That's correct for the main question but not for the bonus.
    $endgroup$
    – MichaelMaggs
    2 days ago


















-1












$begingroup$

TL; DR:




Bob cannot force a win on 12, but has a nonlosing strategy.




We use a notation X-> Y to indicate that a move from number X to number Y is possible, and X->Win to indicate that the current player can force a win if the current number is X. Moving to a winning move is a losing move, as then the other player can win. Similarly, we have X-> Lose.



With this, we build a partial game tree. Formatting was a bit of an adventure, so... sorry for the mess?



Base cases:




Based no Gareth's (correct) answer, we have

1->Win

2->Lose

3->Win

4->Win

5->Lose

6->Win

7->Lose

8->Win

9->Win

10->Win

11->Win

14->Win




Turn 1:




The cubes and squares less than 12 are: 1, 4, 8, 9

We can infer the moves

12->11

12->8

12->4

12->3

All losing moves, which can be disregarded (as a rational player will ignore them)

Also, 12-> 24, which we will eventually find forces the next player to lose or enter a loop.




Turn 2:




Carrying on in this fashion to build a more complete game tree,

24-> 8 (Losing)

24-> 15 (Losing- see below)

24-> 16 (Losing, next player can remove 16)

24-> 20 (Losing, see below)

24-> 23 (Losing- see below)

24-> 48 (Loops to self or 12 in 2 moves, or to 12 in 1 move)

As all these moves are losing except for 48, the active player must move to 48




Turn 3:




15-> 7 (Win- Having found a winning move, we can infer that 15 is a winning position and terminate this branch)


20-> 19 (Winning- see below)


23 -> 7 (Winning)

We can subtract 1,4,8,9,16,25,27, or 36 from 48, allowing move

48 -> 12 (Loop, changing turn parity)
48 -> 39 (Loops to 12 in 1 move)
48 -> 40 (Loops to 24 in 1 move)
48 -> 44 (Losing)
48 -> 47 (Losing)




Turn 4:




19-> 3 (Losing)

19-> 10 (Losing)

19->11 (Losing)

19-> 15 (Losing)

19 -> 18 (Losing- see below)

19 -> 38 (Losing- see below)

All moves here are losing, so moving to 19 is a Winning move.

21 -> 5 (Winning)

23 -> 7 (Winning)

32 -> 7 (Winning)

39 -> 12 (Loop, retaining player parity)
40 -> 24 (Loop, changing player parity)

44 -> 17 (Win)

47 -> 5 (Win)




Turn 5:




We can subtract 1, 4, 8, 9, 16, 25, 27, or 36 from 38, allowing move

38 -> 2 (Winning)


18-> 2 (Winning)

13-> 5 (Winning)
15-> 7 (Winning)
17-> 1 (L)
17 -> 8 (L)
17 -> 9 (L)
17 -> 13 (L)
17 -> 16 (L)
17 -> 34 (L)




Turn 6:




34-> 7 (W)







share|improve this answer











$endgroup$








  • 2




    $begingroup$
    $48 - 19 = 29$.
    $endgroup$
    – noedne
    May 1 at 18:07










  • $begingroup$
    Curses, foiled by arithmatic. I'll see if I can fix this real quick.
    $endgroup$
    – Monk
    May 1 at 18:09











Your Answer








StackExchange.ready(function()
var channelOptions =
tags: "".split(" "),
id: "559"
;
initTagRenderer("".split(" "), "".split(" "), channelOptions);

StackExchange.using("externalEditor", function()
// Have to fire editor after snippets, if snippets enabled
if (StackExchange.settings.snippets.snippetsEnabled)
StackExchange.using("snippets", function()
createEditor();
);

else
createEditor();

);

function createEditor()
StackExchange.prepareEditor(
heartbeatType: 'answer',
autoActivateHeartbeat: false,
convertImagesToLinks: false,
noModals: true,
showLowRepImageUploadWarning: true,
reputationToPostImages: null,
bindNavPrevention: true,
postfix: "",
imageUploader:
brandingHtml: "Powered by u003ca class="icon-imgur-white" href="https://imgur.com/"u003eu003c/au003e",
contentPolicyHtml: "User contributions licensed under u003ca href="https://creativecommons.org/licenses/by-sa/3.0/"u003ecc by-sa 3.0 with attribution requiredu003c/au003e u003ca href="https://stackoverflow.com/legal/content-policy"u003e(content policy)u003c/au003e",
allowUrls: true
,
noCode: true, onDemand: true,
discardSelector: ".discard-answer"
,immediatelyShowMarkdownHelp:true
);



);













draft saved

draft discarded


















StackExchange.ready(
function ()
StackExchange.openid.initPostLogin('.new-post-login', 'https%3a%2f%2fpuzzling.stackexchange.com%2fquestions%2f83458%2fdouble-or-take-game%23new-answer', 'question_page');

);

Post as a guest















Required, but never shown

























3 Answers
3






active

oldest

votes








3 Answers
3






active

oldest

votes









active

oldest

votes






active

oldest

votes









8












$begingroup$

Answer to the non-bonus part:




Next-player winning positions -- i.e., ones where Bob wins if he starts -- from 1 to 11 inclusive are 1,3,4,6,8,9,10,11. Next-player losing positions are 2,5,7. So Alice (who plays second) wins if she chooses 2, 5, or 7, but not if she chooses any other number from 1 to 11.




Proof:




Call a position "winning" or "losing" according to what happens with best play to the player whose move it is. (Or "drawing" if neither player can force a win and at least one can force the game to continue for ever.) Then we find, successively:

0 is losing (whoever just moved there already won the game).

1,4,8,9 are winning (you can move to 0 from any of them).

2 is losing (you can move from there only to 1,4, both of which are winning).

3,6,10 are winning (you can move to 2 from any of them).

5 is losing (you can move from there only to 1,4,10, all of which are winning).

14 is winning (you can move to 5 from it).

7 is losing (you can move from there only to 3,6,14, all of which are winning).

11 is winning (you can move to 7 from it).




Bonus part: From 12




it turns out -- I calculated this by computer -- that the sort of reasoning above is enough to determine that all positions you can get to from 12 other than 24 are next-player wins, and all positions you can get to from 24 other than 48 are next-player wins. Now, we have $12rightarrow24rightarrow48rightarrow12$. So, if Bob loses starting from 12 in particular he must lose when he moves to 24; if Alice wins starting at 24 then it must be by moving to 48 (since all her other moves from 24 lose); if Bob loses when starting at 48 then in particular he must lose by moving to 12, i.e., Alice must win when starting from 12. But this is a contradiction since we started out by supposing that 12 is a loss for the player moving there.




Hence




Alice cannot be sure of winning if she makes Bob move first from 12. He will move to 24, Alice definitely loses if she moves to anything other than 48, and then either Bob can win by doing something other than moving back to 12 (in which case he should) or he can't (in which case he should move back to 12, and then the best Alice can do is to go around the same loop again).







share|improve this answer











$endgroup$












  • $begingroup$
    I didn’t look at this answer when composing mine below, but I came to the same conclusion as @MichaelMaggs. Sorry, Gareth!
    $endgroup$
    – El-Guest
    May 1 at 15:40










  • $begingroup$
    @El-Guest Not sure what you're apologizing for.
    $endgroup$
    – Gareth McCaughan
    May 1 at 15:41










  • $begingroup$
    @GarethMcCaughan for (potentially) nipping your answer.
    $endgroup$
    – El-Guest
    May 1 at 15:45










  • $begingroup$
    Oh! I confess it hadn't occurred to me that that was a danger; I had an essentially correct answer some time before yours, after all. But of course OP can award the checkmark on whatever basis he pleases...
    $endgroup$
    – Gareth McCaughan
    May 1 at 15:50






  • 1




    $begingroup$
    ... I've thrown away the bits of paper on which I did some hand calculations and restarted the computer on which I did some not-hand calculations without saving the relevant stuff. I might do it again and see whether indeed the other moves from 24 can be eliminated -- I think they might be.
    $endgroup$
    – Gareth McCaughan
    May 7 at 8:33















8












$begingroup$

Answer to the non-bonus part:




Next-player winning positions -- i.e., ones where Bob wins if he starts -- from 1 to 11 inclusive are 1,3,4,6,8,9,10,11. Next-player losing positions are 2,5,7. So Alice (who plays second) wins if she chooses 2, 5, or 7, but not if she chooses any other number from 1 to 11.




Proof:




Call a position "winning" or "losing" according to what happens with best play to the player whose move it is. (Or "drawing" if neither player can force a win and at least one can force the game to continue for ever.) Then we find, successively:

0 is losing (whoever just moved there already won the game).

1,4,8,9 are winning (you can move to 0 from any of them).

2 is losing (you can move from there only to 1,4, both of which are winning).

3,6,10 are winning (you can move to 2 from any of them).

5 is losing (you can move from there only to 1,4,10, all of which are winning).

14 is winning (you can move to 5 from it).

7 is losing (you can move from there only to 3,6,14, all of which are winning).

11 is winning (you can move to 7 from it).




Bonus part: From 12




it turns out -- I calculated this by computer -- that the sort of reasoning above is enough to determine that all positions you can get to from 12 other than 24 are next-player wins, and all positions you can get to from 24 other than 48 are next-player wins. Now, we have $12rightarrow24rightarrow48rightarrow12$. So, if Bob loses starting from 12 in particular he must lose when he moves to 24; if Alice wins starting at 24 then it must be by moving to 48 (since all her other moves from 24 lose); if Bob loses when starting at 48 then in particular he must lose by moving to 12, i.e., Alice must win when starting from 12. But this is a contradiction since we started out by supposing that 12 is a loss for the player moving there.




Hence




Alice cannot be sure of winning if she makes Bob move first from 12. He will move to 24, Alice definitely loses if she moves to anything other than 48, and then either Bob can win by doing something other than moving back to 12 (in which case he should) or he can't (in which case he should move back to 12, and then the best Alice can do is to go around the same loop again).







share|improve this answer











$endgroup$












  • $begingroup$
    I didn’t look at this answer when composing mine below, but I came to the same conclusion as @MichaelMaggs. Sorry, Gareth!
    $endgroup$
    – El-Guest
    May 1 at 15:40










  • $begingroup$
    @El-Guest Not sure what you're apologizing for.
    $endgroup$
    – Gareth McCaughan
    May 1 at 15:41










  • $begingroup$
    @GarethMcCaughan for (potentially) nipping your answer.
    $endgroup$
    – El-Guest
    May 1 at 15:45










  • $begingroup$
    Oh! I confess it hadn't occurred to me that that was a danger; I had an essentially correct answer some time before yours, after all. But of course OP can award the checkmark on whatever basis he pleases...
    $endgroup$
    – Gareth McCaughan
    May 1 at 15:50






  • 1




    $begingroup$
    ... I've thrown away the bits of paper on which I did some hand calculations and restarted the computer on which I did some not-hand calculations without saving the relevant stuff. I might do it again and see whether indeed the other moves from 24 can be eliminated -- I think they might be.
    $endgroup$
    – Gareth McCaughan
    May 7 at 8:33













8












8








8





$begingroup$

Answer to the non-bonus part:




Next-player winning positions -- i.e., ones where Bob wins if he starts -- from 1 to 11 inclusive are 1,3,4,6,8,9,10,11. Next-player losing positions are 2,5,7. So Alice (who plays second) wins if she chooses 2, 5, or 7, but not if she chooses any other number from 1 to 11.




Proof:




Call a position "winning" or "losing" according to what happens with best play to the player whose move it is. (Or "drawing" if neither player can force a win and at least one can force the game to continue for ever.) Then we find, successively:

0 is losing (whoever just moved there already won the game).

1,4,8,9 are winning (you can move to 0 from any of them).

2 is losing (you can move from there only to 1,4, both of which are winning).

3,6,10 are winning (you can move to 2 from any of them).

5 is losing (you can move from there only to 1,4,10, all of which are winning).

14 is winning (you can move to 5 from it).

7 is losing (you can move from there only to 3,6,14, all of which are winning).

11 is winning (you can move to 7 from it).




Bonus part: From 12




it turns out -- I calculated this by computer -- that the sort of reasoning above is enough to determine that all positions you can get to from 12 other than 24 are next-player wins, and all positions you can get to from 24 other than 48 are next-player wins. Now, we have $12rightarrow24rightarrow48rightarrow12$. So, if Bob loses starting from 12 in particular he must lose when he moves to 24; if Alice wins starting at 24 then it must be by moving to 48 (since all her other moves from 24 lose); if Bob loses when starting at 48 then in particular he must lose by moving to 12, i.e., Alice must win when starting from 12. But this is a contradiction since we started out by supposing that 12 is a loss for the player moving there.




Hence




Alice cannot be sure of winning if she makes Bob move first from 12. He will move to 24, Alice definitely loses if she moves to anything other than 48, and then either Bob can win by doing something other than moving back to 12 (in which case he should) or he can't (in which case he should move back to 12, and then the best Alice can do is to go around the same loop again).







share|improve this answer











$endgroup$



Answer to the non-bonus part:




Next-player winning positions -- i.e., ones where Bob wins if he starts -- from 1 to 11 inclusive are 1,3,4,6,8,9,10,11. Next-player losing positions are 2,5,7. So Alice (who plays second) wins if she chooses 2, 5, or 7, but not if she chooses any other number from 1 to 11.




Proof:




Call a position "winning" or "losing" according to what happens with best play to the player whose move it is. (Or "drawing" if neither player can force a win and at least one can force the game to continue for ever.) Then we find, successively:

0 is losing (whoever just moved there already won the game).

1,4,8,9 are winning (you can move to 0 from any of them).

2 is losing (you can move from there only to 1,4, both of which are winning).

3,6,10 are winning (you can move to 2 from any of them).

5 is losing (you can move from there only to 1,4,10, all of which are winning).

14 is winning (you can move to 5 from it).

7 is losing (you can move from there only to 3,6,14, all of which are winning).

11 is winning (you can move to 7 from it).




Bonus part: From 12




it turns out -- I calculated this by computer -- that the sort of reasoning above is enough to determine that all positions you can get to from 12 other than 24 are next-player wins, and all positions you can get to from 24 other than 48 are next-player wins. Now, we have $12rightarrow24rightarrow48rightarrow12$. So, if Bob loses starting from 12 in particular he must lose when he moves to 24; if Alice wins starting at 24 then it must be by moving to 48 (since all her other moves from 24 lose); if Bob loses when starting at 48 then in particular he must lose by moving to 12, i.e., Alice must win when starting from 12. But this is a contradiction since we started out by supposing that 12 is a loss for the player moving there.




Hence




Alice cannot be sure of winning if she makes Bob move first from 12. He will move to 24, Alice definitely loses if she moves to anything other than 48, and then either Bob can win by doing something other than moving back to 12 (in which case he should) or he can't (in which case he should move back to 12, and then the best Alice can do is to go around the same loop again).








share|improve this answer














share|improve this answer



share|improve this answer








edited 2 days ago

























answered May 1 at 15:34









Gareth McCaughanGareth McCaughan

71k3179276




71k3179276











  • $begingroup$
    I didn’t look at this answer when composing mine below, but I came to the same conclusion as @MichaelMaggs. Sorry, Gareth!
    $endgroup$
    – El-Guest
    May 1 at 15:40










  • $begingroup$
    @El-Guest Not sure what you're apologizing for.
    $endgroup$
    – Gareth McCaughan
    May 1 at 15:41










  • $begingroup$
    @GarethMcCaughan for (potentially) nipping your answer.
    $endgroup$
    – El-Guest
    May 1 at 15:45










  • $begingroup$
    Oh! I confess it hadn't occurred to me that that was a danger; I had an essentially correct answer some time before yours, after all. But of course OP can award the checkmark on whatever basis he pleases...
    $endgroup$
    – Gareth McCaughan
    May 1 at 15:50






  • 1




    $begingroup$
    ... I've thrown away the bits of paper on which I did some hand calculations and restarted the computer on which I did some not-hand calculations without saving the relevant stuff. I might do it again and see whether indeed the other moves from 24 can be eliminated -- I think they might be.
    $endgroup$
    – Gareth McCaughan
    May 7 at 8:33
















  • $begingroup$
    I didn’t look at this answer when composing mine below, but I came to the same conclusion as @MichaelMaggs. Sorry, Gareth!
    $endgroup$
    – El-Guest
    May 1 at 15:40










  • $begingroup$
    @El-Guest Not sure what you're apologizing for.
    $endgroup$
    – Gareth McCaughan
    May 1 at 15:41










  • $begingroup$
    @GarethMcCaughan for (potentially) nipping your answer.
    $endgroup$
    – El-Guest
    May 1 at 15:45










  • $begingroup$
    Oh! I confess it hadn't occurred to me that that was a danger; I had an essentially correct answer some time before yours, after all. But of course OP can award the checkmark on whatever basis he pleases...
    $endgroup$
    – Gareth McCaughan
    May 1 at 15:50






  • 1




    $begingroup$
    ... I've thrown away the bits of paper on which I did some hand calculations and restarted the computer on which I did some not-hand calculations without saving the relevant stuff. I might do it again and see whether indeed the other moves from 24 can be eliminated -- I think they might be.
    $endgroup$
    – Gareth McCaughan
    May 7 at 8:33















$begingroup$
I didn’t look at this answer when composing mine below, but I came to the same conclusion as @MichaelMaggs. Sorry, Gareth!
$endgroup$
– El-Guest
May 1 at 15:40




$begingroup$
I didn’t look at this answer when composing mine below, but I came to the same conclusion as @MichaelMaggs. Sorry, Gareth!
$endgroup$
– El-Guest
May 1 at 15:40












$begingroup$
@El-Guest Not sure what you're apologizing for.
$endgroup$
– Gareth McCaughan
May 1 at 15:41




$begingroup$
@El-Guest Not sure what you're apologizing for.
$endgroup$
– Gareth McCaughan
May 1 at 15:41












$begingroup$
@GarethMcCaughan for (potentially) nipping your answer.
$endgroup$
– El-Guest
May 1 at 15:45




$begingroup$
@GarethMcCaughan for (potentially) nipping your answer.
$endgroup$
– El-Guest
May 1 at 15:45












$begingroup$
Oh! I confess it hadn't occurred to me that that was a danger; I had an essentially correct answer some time before yours, after all. But of course OP can award the checkmark on whatever basis he pleases...
$endgroup$
– Gareth McCaughan
May 1 at 15:50




$begingroup$
Oh! I confess it hadn't occurred to me that that was a danger; I had an essentially correct answer some time before yours, after all. But of course OP can award the checkmark on whatever basis he pleases...
$endgroup$
– Gareth McCaughan
May 1 at 15:50




1




1




$begingroup$
... I've thrown away the bits of paper on which I did some hand calculations and restarted the computer on which I did some not-hand calculations without saving the relevant stuff. I might do it again and see whether indeed the other moves from 24 can be eliminated -- I think they might be.
$endgroup$
– Gareth McCaughan
May 7 at 8:33




$begingroup$
... I've thrown away the bits of paper on which I did some hand calculations and restarted the computer on which I did some not-hand calculations without saving the relevant stuff. I might do it again and see whether indeed the other moves from 24 can be eliminated -- I think they might be.
$endgroup$
– Gareth McCaughan
May 7 at 8:33











4












$begingroup$

Let’s see: I assume that all numbers must be greater than or equal to zero, otherwise the possibilities become too painful. If that was the intention, I’ll look harder.



1




Bob subtracts 1, so Alice loses.




2




Bob can’t double, because that leads to 4 and a Bob-loss; he can’t subtract 1 because that leads to 1 and a Bob-loss. Therefore 2 is a winning number for Alice.




3




Bob subtracts 1, forcing a loss for Alice as above.




4




Bob subtracts 4 and Alice loses.




5




Bob can’t subtract either 1 or 4, since Alice can win. He also can’t double, because that makes 10 — then Alice subtracts 8 forcing a Bob-loss. Therefore 5 is a winning number for Alice.




6




Bob subtracts 4, forcing a loss for Alice as 2 is left.




7




If Bob subtracts 1, Alice subtracts 4, forcing a loss for Bob. If Bob subtracts 4, Alice subtracts 1, forcing a loss for Bob. Bob therefore has to double to 14. But Bob doubling to 14 means Alice can subtract 9 to get to 5, which forces a Bob-loss. Thus 7 is a winning number for Alice.




8




Bob subtracts 8 ($2^3$) and Alice loses.




9




Bob subtracts 4, forcing a loss for Alice as 5 is left.




10




Bob subtracts 8, leading to an Alice loss.




11




Bob subtracts 4, leading to an Alice loss.




Therefore the only starting numbers that win for Alice are




2, 5, and 7.




Bonus Part: 12




Bob has to double to 24 because he can’t get to 2, 5, or 7. Then Alice can’t subtract 1, because it’ll leave Bob with 23; he can subtract 16 and win. Alice can’t subtract 9, because it’ll leave Bob with 15; he can subtract 8 and win. Alice can’t subtract 8, because it’ll leave Bob with 16; he can subtract 9 and win. Alice can’t subtract 16, because it’ll leave Bob with 8; he can subtract 1 and win. Doubling here allows Bob to subtract 36 and leave Alice with 12. It appears then that this could create a stable loop solution where neither player wins. If Alice subtracts 4, Bob is left with 20. Bob can’t subtract 4, because leaving 16 allows Alice to subtract 9 and get to 7. Bob can’t subtract 9, because leaving 11 allows Alice to subtract 4 and get to 7. Bob can’t subtract 16 because it leaves 4 for Alice. However, subtracting 8 leaves an equilibrium standoff solution. This might be optimal...?







share|improve this answer











$endgroup$












  • $begingroup$
    That's correct for the main question but not for the bonus.
    $endgroup$
    – MichaelMaggs
    2 days ago















4












$begingroup$

Let’s see: I assume that all numbers must be greater than or equal to zero, otherwise the possibilities become too painful. If that was the intention, I’ll look harder.



1




Bob subtracts 1, so Alice loses.




2




Bob can’t double, because that leads to 4 and a Bob-loss; he can’t subtract 1 because that leads to 1 and a Bob-loss. Therefore 2 is a winning number for Alice.




3




Bob subtracts 1, forcing a loss for Alice as above.




4




Bob subtracts 4 and Alice loses.




5




Bob can’t subtract either 1 or 4, since Alice can win. He also can’t double, because that makes 10 — then Alice subtracts 8 forcing a Bob-loss. Therefore 5 is a winning number for Alice.




6




Bob subtracts 4, forcing a loss for Alice as 2 is left.




7




If Bob subtracts 1, Alice subtracts 4, forcing a loss for Bob. If Bob subtracts 4, Alice subtracts 1, forcing a loss for Bob. Bob therefore has to double to 14. But Bob doubling to 14 means Alice can subtract 9 to get to 5, which forces a Bob-loss. Thus 7 is a winning number for Alice.




8




Bob subtracts 8 ($2^3$) and Alice loses.




9




Bob subtracts 4, forcing a loss for Alice as 5 is left.




10




Bob subtracts 8, leading to an Alice loss.




11




Bob subtracts 4, leading to an Alice loss.




Therefore the only starting numbers that win for Alice are




2, 5, and 7.




Bonus Part: 12




Bob has to double to 24 because he can’t get to 2, 5, or 7. Then Alice can’t subtract 1, because it’ll leave Bob with 23; he can subtract 16 and win. Alice can’t subtract 9, because it’ll leave Bob with 15; he can subtract 8 and win. Alice can’t subtract 8, because it’ll leave Bob with 16; he can subtract 9 and win. Alice can’t subtract 16, because it’ll leave Bob with 8; he can subtract 1 and win. Doubling here allows Bob to subtract 36 and leave Alice with 12. It appears then that this could create a stable loop solution where neither player wins. If Alice subtracts 4, Bob is left with 20. Bob can’t subtract 4, because leaving 16 allows Alice to subtract 9 and get to 7. Bob can’t subtract 9, because leaving 11 allows Alice to subtract 4 and get to 7. Bob can’t subtract 16 because it leaves 4 for Alice. However, subtracting 8 leaves an equilibrium standoff solution. This might be optimal...?







share|improve this answer











$endgroup$












  • $begingroup$
    That's correct for the main question but not for the bonus.
    $endgroup$
    – MichaelMaggs
    2 days ago













4












4








4





$begingroup$

Let’s see: I assume that all numbers must be greater than or equal to zero, otherwise the possibilities become too painful. If that was the intention, I’ll look harder.



1




Bob subtracts 1, so Alice loses.




2




Bob can’t double, because that leads to 4 and a Bob-loss; he can’t subtract 1 because that leads to 1 and a Bob-loss. Therefore 2 is a winning number for Alice.




3




Bob subtracts 1, forcing a loss for Alice as above.




4




Bob subtracts 4 and Alice loses.




5




Bob can’t subtract either 1 or 4, since Alice can win. He also can’t double, because that makes 10 — then Alice subtracts 8 forcing a Bob-loss. Therefore 5 is a winning number for Alice.




6




Bob subtracts 4, forcing a loss for Alice as 2 is left.




7




If Bob subtracts 1, Alice subtracts 4, forcing a loss for Bob. If Bob subtracts 4, Alice subtracts 1, forcing a loss for Bob. Bob therefore has to double to 14. But Bob doubling to 14 means Alice can subtract 9 to get to 5, which forces a Bob-loss. Thus 7 is a winning number for Alice.




8




Bob subtracts 8 ($2^3$) and Alice loses.




9




Bob subtracts 4, forcing a loss for Alice as 5 is left.




10




Bob subtracts 8, leading to an Alice loss.




11




Bob subtracts 4, leading to an Alice loss.




Therefore the only starting numbers that win for Alice are




2, 5, and 7.




Bonus Part: 12




Bob has to double to 24 because he can’t get to 2, 5, or 7. Then Alice can’t subtract 1, because it’ll leave Bob with 23; he can subtract 16 and win. Alice can’t subtract 9, because it’ll leave Bob with 15; he can subtract 8 and win. Alice can’t subtract 8, because it’ll leave Bob with 16; he can subtract 9 and win. Alice can’t subtract 16, because it’ll leave Bob with 8; he can subtract 1 and win. Doubling here allows Bob to subtract 36 and leave Alice with 12. It appears then that this could create a stable loop solution where neither player wins. If Alice subtracts 4, Bob is left with 20. Bob can’t subtract 4, because leaving 16 allows Alice to subtract 9 and get to 7. Bob can’t subtract 9, because leaving 11 allows Alice to subtract 4 and get to 7. Bob can’t subtract 16 because it leaves 4 for Alice. However, subtracting 8 leaves an equilibrium standoff solution. This might be optimal...?







share|improve this answer











$endgroup$



Let’s see: I assume that all numbers must be greater than or equal to zero, otherwise the possibilities become too painful. If that was the intention, I’ll look harder.



1




Bob subtracts 1, so Alice loses.




2




Bob can’t double, because that leads to 4 and a Bob-loss; he can’t subtract 1 because that leads to 1 and a Bob-loss. Therefore 2 is a winning number for Alice.




3




Bob subtracts 1, forcing a loss for Alice as above.




4




Bob subtracts 4 and Alice loses.




5




Bob can’t subtract either 1 or 4, since Alice can win. He also can’t double, because that makes 10 — then Alice subtracts 8 forcing a Bob-loss. Therefore 5 is a winning number for Alice.




6




Bob subtracts 4, forcing a loss for Alice as 2 is left.




7




If Bob subtracts 1, Alice subtracts 4, forcing a loss for Bob. If Bob subtracts 4, Alice subtracts 1, forcing a loss for Bob. Bob therefore has to double to 14. But Bob doubling to 14 means Alice can subtract 9 to get to 5, which forces a Bob-loss. Thus 7 is a winning number for Alice.




8




Bob subtracts 8 ($2^3$) and Alice loses.




9




Bob subtracts 4, forcing a loss for Alice as 5 is left.




10




Bob subtracts 8, leading to an Alice loss.




11




Bob subtracts 4, leading to an Alice loss.




Therefore the only starting numbers that win for Alice are




2, 5, and 7.




Bonus Part: 12




Bob has to double to 24 because he can’t get to 2, 5, or 7. Then Alice can’t subtract 1, because it’ll leave Bob with 23; he can subtract 16 and win. Alice can’t subtract 9, because it’ll leave Bob with 15; he can subtract 8 and win. Alice can’t subtract 8, because it’ll leave Bob with 16; he can subtract 9 and win. Alice can’t subtract 16, because it’ll leave Bob with 8; he can subtract 1 and win. Doubling here allows Bob to subtract 36 and leave Alice with 12. It appears then that this could create a stable loop solution where neither player wins. If Alice subtracts 4, Bob is left with 20. Bob can’t subtract 4, because leaving 16 allows Alice to subtract 9 and get to 7. Bob can’t subtract 9, because leaving 11 allows Alice to subtract 4 and get to 7. Bob can’t subtract 16 because it leaves 4 for Alice. However, subtracting 8 leaves an equilibrium standoff solution. This might be optimal...?








share|improve this answer














share|improve this answer



share|improve this answer








edited May 1 at 16:33

























answered May 1 at 15:39









El-GuestEl-Guest

22.6k35295




22.6k35295











  • $begingroup$
    That's correct for the main question but not for the bonus.
    $endgroup$
    – MichaelMaggs
    2 days ago
















  • $begingroup$
    That's correct for the main question but not for the bonus.
    $endgroup$
    – MichaelMaggs
    2 days ago















$begingroup$
That's correct for the main question but not for the bonus.
$endgroup$
– MichaelMaggs
2 days ago




$begingroup$
That's correct for the main question but not for the bonus.
$endgroup$
– MichaelMaggs
2 days ago











-1












$begingroup$

TL; DR:




Bob cannot force a win on 12, but has a nonlosing strategy.




We use a notation X-> Y to indicate that a move from number X to number Y is possible, and X->Win to indicate that the current player can force a win if the current number is X. Moving to a winning move is a losing move, as then the other player can win. Similarly, we have X-> Lose.



With this, we build a partial game tree. Formatting was a bit of an adventure, so... sorry for the mess?



Base cases:




Based no Gareth's (correct) answer, we have

1->Win

2->Lose

3->Win

4->Win

5->Lose

6->Win

7->Lose

8->Win

9->Win

10->Win

11->Win

14->Win




Turn 1:




The cubes and squares less than 12 are: 1, 4, 8, 9

We can infer the moves

12->11

12->8

12->4

12->3

All losing moves, which can be disregarded (as a rational player will ignore them)

Also, 12-> 24, which we will eventually find forces the next player to lose or enter a loop.




Turn 2:




Carrying on in this fashion to build a more complete game tree,

24-> 8 (Losing)

24-> 15 (Losing- see below)

24-> 16 (Losing, next player can remove 16)

24-> 20 (Losing, see below)

24-> 23 (Losing- see below)

24-> 48 (Loops to self or 12 in 2 moves, or to 12 in 1 move)

As all these moves are losing except for 48, the active player must move to 48




Turn 3:




15-> 7 (Win- Having found a winning move, we can infer that 15 is a winning position and terminate this branch)


20-> 19 (Winning- see below)


23 -> 7 (Winning)

We can subtract 1,4,8,9,16,25,27, or 36 from 48, allowing move

48 -> 12 (Loop, changing turn parity)
48 -> 39 (Loops to 12 in 1 move)
48 -> 40 (Loops to 24 in 1 move)
48 -> 44 (Losing)
48 -> 47 (Losing)




Turn 4:




19-> 3 (Losing)

19-> 10 (Losing)

19->11 (Losing)

19-> 15 (Losing)

19 -> 18 (Losing- see below)

19 -> 38 (Losing- see below)

All moves here are losing, so moving to 19 is a Winning move.

21 -> 5 (Winning)

23 -> 7 (Winning)

32 -> 7 (Winning)

39 -> 12 (Loop, retaining player parity)
40 -> 24 (Loop, changing player parity)

44 -> 17 (Win)

47 -> 5 (Win)




Turn 5:




We can subtract 1, 4, 8, 9, 16, 25, 27, or 36 from 38, allowing move

38 -> 2 (Winning)


18-> 2 (Winning)

13-> 5 (Winning)
15-> 7 (Winning)
17-> 1 (L)
17 -> 8 (L)
17 -> 9 (L)
17 -> 13 (L)
17 -> 16 (L)
17 -> 34 (L)




Turn 6:




34-> 7 (W)







share|improve this answer











$endgroup$








  • 2




    $begingroup$
    $48 - 19 = 29$.
    $endgroup$
    – noedne
    May 1 at 18:07










  • $begingroup$
    Curses, foiled by arithmatic. I'll see if I can fix this real quick.
    $endgroup$
    – Monk
    May 1 at 18:09















-1












$begingroup$

TL; DR:




Bob cannot force a win on 12, but has a nonlosing strategy.




We use a notation X-> Y to indicate that a move from number X to number Y is possible, and X->Win to indicate that the current player can force a win if the current number is X. Moving to a winning move is a losing move, as then the other player can win. Similarly, we have X-> Lose.



With this, we build a partial game tree. Formatting was a bit of an adventure, so... sorry for the mess?



Base cases:




Based no Gareth's (correct) answer, we have

1->Win

2->Lose

3->Win

4->Win

5->Lose

6->Win

7->Lose

8->Win

9->Win

10->Win

11->Win

14->Win




Turn 1:




The cubes and squares less than 12 are: 1, 4, 8, 9

We can infer the moves

12->11

12->8

12->4

12->3

All losing moves, which can be disregarded (as a rational player will ignore them)

Also, 12-> 24, which we will eventually find forces the next player to lose or enter a loop.




Turn 2:




Carrying on in this fashion to build a more complete game tree,

24-> 8 (Losing)

24-> 15 (Losing- see below)

24-> 16 (Losing, next player can remove 16)

24-> 20 (Losing, see below)

24-> 23 (Losing- see below)

24-> 48 (Loops to self or 12 in 2 moves, or to 12 in 1 move)

As all these moves are losing except for 48, the active player must move to 48




Turn 3:




15-> 7 (Win- Having found a winning move, we can infer that 15 is a winning position and terminate this branch)


20-> 19 (Winning- see below)


23 -> 7 (Winning)

We can subtract 1,4,8,9,16,25,27, or 36 from 48, allowing move

48 -> 12 (Loop, changing turn parity)
48 -> 39 (Loops to 12 in 1 move)
48 -> 40 (Loops to 24 in 1 move)
48 -> 44 (Losing)
48 -> 47 (Losing)




Turn 4:




19-> 3 (Losing)

19-> 10 (Losing)

19->11 (Losing)

19-> 15 (Losing)

19 -> 18 (Losing- see below)

19 -> 38 (Losing- see below)

All moves here are losing, so moving to 19 is a Winning move.

21 -> 5 (Winning)

23 -> 7 (Winning)

32 -> 7 (Winning)

39 -> 12 (Loop, retaining player parity)
40 -> 24 (Loop, changing player parity)

44 -> 17 (Win)

47 -> 5 (Win)




Turn 5:




We can subtract 1, 4, 8, 9, 16, 25, 27, or 36 from 38, allowing move

38 -> 2 (Winning)


18-> 2 (Winning)

13-> 5 (Winning)
15-> 7 (Winning)
17-> 1 (L)
17 -> 8 (L)
17 -> 9 (L)
17 -> 13 (L)
17 -> 16 (L)
17 -> 34 (L)




Turn 6:




34-> 7 (W)







share|improve this answer











$endgroup$








  • 2




    $begingroup$
    $48 - 19 = 29$.
    $endgroup$
    – noedne
    May 1 at 18:07










  • $begingroup$
    Curses, foiled by arithmatic. I'll see if I can fix this real quick.
    $endgroup$
    – Monk
    May 1 at 18:09













-1












-1








-1





$begingroup$

TL; DR:




Bob cannot force a win on 12, but has a nonlosing strategy.




We use a notation X-> Y to indicate that a move from number X to number Y is possible, and X->Win to indicate that the current player can force a win if the current number is X. Moving to a winning move is a losing move, as then the other player can win. Similarly, we have X-> Lose.



With this, we build a partial game tree. Formatting was a bit of an adventure, so... sorry for the mess?



Base cases:




Based no Gareth's (correct) answer, we have

1->Win

2->Lose

3->Win

4->Win

5->Lose

6->Win

7->Lose

8->Win

9->Win

10->Win

11->Win

14->Win




Turn 1:




The cubes and squares less than 12 are: 1, 4, 8, 9

We can infer the moves

12->11

12->8

12->4

12->3

All losing moves, which can be disregarded (as a rational player will ignore them)

Also, 12-> 24, which we will eventually find forces the next player to lose or enter a loop.




Turn 2:




Carrying on in this fashion to build a more complete game tree,

24-> 8 (Losing)

24-> 15 (Losing- see below)

24-> 16 (Losing, next player can remove 16)

24-> 20 (Losing, see below)

24-> 23 (Losing- see below)

24-> 48 (Loops to self or 12 in 2 moves, or to 12 in 1 move)

As all these moves are losing except for 48, the active player must move to 48




Turn 3:




15-> 7 (Win- Having found a winning move, we can infer that 15 is a winning position and terminate this branch)


20-> 19 (Winning- see below)


23 -> 7 (Winning)

We can subtract 1,4,8,9,16,25,27, or 36 from 48, allowing move

48 -> 12 (Loop, changing turn parity)
48 -> 39 (Loops to 12 in 1 move)
48 -> 40 (Loops to 24 in 1 move)
48 -> 44 (Losing)
48 -> 47 (Losing)




Turn 4:




19-> 3 (Losing)

19-> 10 (Losing)

19->11 (Losing)

19-> 15 (Losing)

19 -> 18 (Losing- see below)

19 -> 38 (Losing- see below)

All moves here are losing, so moving to 19 is a Winning move.

21 -> 5 (Winning)

23 -> 7 (Winning)

32 -> 7 (Winning)

39 -> 12 (Loop, retaining player parity)
40 -> 24 (Loop, changing player parity)

44 -> 17 (Win)

47 -> 5 (Win)




Turn 5:




We can subtract 1, 4, 8, 9, 16, 25, 27, or 36 from 38, allowing move

38 -> 2 (Winning)


18-> 2 (Winning)

13-> 5 (Winning)
15-> 7 (Winning)
17-> 1 (L)
17 -> 8 (L)
17 -> 9 (L)
17 -> 13 (L)
17 -> 16 (L)
17 -> 34 (L)




Turn 6:




34-> 7 (W)







share|improve this answer











$endgroup$



TL; DR:




Bob cannot force a win on 12, but has a nonlosing strategy.




We use a notation X-> Y to indicate that a move from number X to number Y is possible, and X->Win to indicate that the current player can force a win if the current number is X. Moving to a winning move is a losing move, as then the other player can win. Similarly, we have X-> Lose.



With this, we build a partial game tree. Formatting was a bit of an adventure, so... sorry for the mess?



Base cases:




Based no Gareth's (correct) answer, we have

1->Win

2->Lose

3->Win

4->Win

5->Lose

6->Win

7->Lose

8->Win

9->Win

10->Win

11->Win

14->Win




Turn 1:




The cubes and squares less than 12 are: 1, 4, 8, 9

We can infer the moves

12->11

12->8

12->4

12->3

All losing moves, which can be disregarded (as a rational player will ignore them)

Also, 12-> 24, which we will eventually find forces the next player to lose or enter a loop.




Turn 2:




Carrying on in this fashion to build a more complete game tree,

24-> 8 (Losing)

24-> 15 (Losing- see below)

24-> 16 (Losing, next player can remove 16)

24-> 20 (Losing, see below)

24-> 23 (Losing- see below)

24-> 48 (Loops to self or 12 in 2 moves, or to 12 in 1 move)

As all these moves are losing except for 48, the active player must move to 48




Turn 3:




15-> 7 (Win- Having found a winning move, we can infer that 15 is a winning position and terminate this branch)


20-> 19 (Winning- see below)


23 -> 7 (Winning)

We can subtract 1,4,8,9,16,25,27, or 36 from 48, allowing move

48 -> 12 (Loop, changing turn parity)
48 -> 39 (Loops to 12 in 1 move)
48 -> 40 (Loops to 24 in 1 move)
48 -> 44 (Losing)
48 -> 47 (Losing)




Turn 4:




19-> 3 (Losing)

19-> 10 (Losing)

19->11 (Losing)

19-> 15 (Losing)

19 -> 18 (Losing- see below)

19 -> 38 (Losing- see below)

All moves here are losing, so moving to 19 is a Winning move.

21 -> 5 (Winning)

23 -> 7 (Winning)

32 -> 7 (Winning)

39 -> 12 (Loop, retaining player parity)
40 -> 24 (Loop, changing player parity)

44 -> 17 (Win)

47 -> 5 (Win)




Turn 5:




We can subtract 1, 4, 8, 9, 16, 25, 27, or 36 from 38, allowing move

38 -> 2 (Winning)


18-> 2 (Winning)

13-> 5 (Winning)
15-> 7 (Winning)
17-> 1 (L)
17 -> 8 (L)
17 -> 9 (L)
17 -> 13 (L)
17 -> 16 (L)
17 -> 34 (L)




Turn 6:




34-> 7 (W)








share|improve this answer














share|improve this answer



share|improve this answer








edited May 1 at 19:55

























answered May 1 at 18:01









MonkMonk

6414




6414







  • 2




    $begingroup$
    $48 - 19 = 29$.
    $endgroup$
    – noedne
    May 1 at 18:07










  • $begingroup$
    Curses, foiled by arithmatic. I'll see if I can fix this real quick.
    $endgroup$
    – Monk
    May 1 at 18:09












  • 2




    $begingroup$
    $48 - 19 = 29$.
    $endgroup$
    – noedne
    May 1 at 18:07










  • $begingroup$
    Curses, foiled by arithmatic. I'll see if I can fix this real quick.
    $endgroup$
    – Monk
    May 1 at 18:09







2




2




$begingroup$
$48 - 19 = 29$.
$endgroup$
– noedne
May 1 at 18:07




$begingroup$
$48 - 19 = 29$.
$endgroup$
– noedne
May 1 at 18:07












$begingroup$
Curses, foiled by arithmatic. I'll see if I can fix this real quick.
$endgroup$
– Monk
May 1 at 18:09




$begingroup$
Curses, foiled by arithmatic. I'll see if I can fix this real quick.
$endgroup$
– Monk
May 1 at 18:09

















draft saved

draft discarded
















































Thanks for contributing an answer to Puzzling Stack Exchange!


  • Please be sure to answer the question. Provide details and share your research!

But avoid


  • Asking for help, clarification, or responding to other answers.

  • Making statements based on opinion; back them up with references or personal experience.

Use MathJax to format equations. MathJax reference.


To learn more, see our tips on writing great answers.




draft saved


draft discarded














StackExchange.ready(
function ()
StackExchange.openid.initPostLogin('.new-post-login', 'https%3a%2f%2fpuzzling.stackexchange.com%2fquestions%2f83458%2fdouble-or-take-game%23new-answer', 'question_page');

);

Post as a guest















Required, but never shown





















































Required, but never shown














Required, but never shown












Required, but never shown







Required, but never shown

































Required, but never shown














Required, but never shown












Required, but never shown







Required, but never shown







Popular posts from this blog

Wikipedia:Vital articles Мазмуну Biography - Өмүр баян Philosophy and psychology - Философия жана психология Religion - Дин Social sciences - Коомдук илимдер Language and literature - Тил жана адабият Science - Илим Technology - Технология Arts and recreation - Искусство жана эс алуу History and geography - Тарых жана география Навигация менюсу

Bruxelas-Capital Índice Historia | Composición | Situación lingüística | Clima | Cidades irmandadas | Notas | Véxase tamén | Menú de navegacióneO uso das linguas en Bruxelas e a situación do neerlandés"Rexión de Bruxelas Capital"o orixinalSitio da rexiónPáxina de Bruselas no sitio da Oficina de Promoción Turística de Valonia e BruxelasMapa Interactivo da Rexión de Bruxelas-CapitaleeWorldCat332144929079854441105155190212ID28008674080552-90000 0001 0666 3698n94104302ID540940339365017018237

What should I write in an apology letter, since I have decided not to join a company after accepting an offer letterShould I keep looking after accepting a job offer?What should I do when I've been verbally told I would get an offer letter, but still haven't gotten one after 4 weeks?Do I accept an offer from a company that I am not likely to join?New job hasn't confirmed starting date and I want to give current employer as much notice as possibleHow should I address my manager in my resignation letter?HR delayed background verification, now jobless as resignedNo email communication after accepting a formal written offer. How should I phrase the call?What should I do if after receiving a verbal offer letter I am informed that my written job offer is put on hold due to some internal issues?Should I inform the current employer that I am about to resign within 1-2 weeks since I have signed the offer letter and waiting for visa?What company will do, if I send their offer letter to another company